¿Qué sucede si intenta aplicar las ecuaciones de Maxwell a este sistema mecánico cuántico?

En otra publicación , discutimos la carga oscilante en un átomo de hidrógeno y el peso de la opinión parecía ser que, de hecho, hay una carga oscilante cuando se considera la superposición de los estados 1s y 2p. Uno de los corresponsales (freecharly) fue un poco más allá y dijo que Schroedinger creía que esta carga oscilante era la fuente de radiación. Me pregunto si el cálculo real confirma esto. Específicamente, en el caso del átomo de hidrógeno en esta superposición particular, ¿obtiene los tiempos de decaimiento correctos para la superposición de estados si aplica las ecuaciones de Maxwell a la carga oscilante y supone que a medida que el sistema pierde energía por radiación, la "probabilidad" fluye del estado 2p al 1s de acuerdo con la cantidad de energía que queda en el sistema?

EDITAR: Algunas personas se oponen de diferentes maneras a la premisa básica de la pregunta, así que permítanme hacerlo un poco más específico: no estoy preguntando si los átomos de hidrógeno REALMENTE EXISTEN en una superposición particular de estos estados. (Puedo preguntar eso en otra pregunta). Lo que estoy preguntando aquí es SI toma (solo para ser específico) una superposición 50-50 de los estados 1s y 2p, y aplica las ecuaciones de Maxwell a la carga oscilante, Y asume que a medida que el átomo irradia, la probabilidad se drena del estado excitado al estado fundamental de tal manera que se mantiene la conservación de la energía... SI haces todas esas cosas, ¿obtienes un resultado consistente con el QM estándar?

Tienes otro problema: si solo inicializas en el 2 pag estado, que debería darte la emisión más fuerte, no hay oscilación de carga y, por lo tanto, no hay radiación. (Del mismo modo, si inicializa, por ejemplo, 0.01 | 1 s + 0.99 | 2 pag , la amplitud de la oscilación será muy pequeña y llevará mucho tiempo alcanzar la velocidad). Dicho esto, esta pregunta mejoraría bastante si especifica el mecanismo que tiene en mente para "a medida que el sistema pierde energía, el la probabilidad fluye del estado 2p al estado 2s". ¿Estás asumiendo la dinámica de Schrödinger? ¿O algo mas?
El punto discutible, Marty, es que el átomo de hidrógeno no está realmente en una superposición de estados 1s y 2p, no está en ningún estado. Es similar a un estado de 1s que se mueve de un lado a otro con un valor de energía de 10,2 ev, pero en realidad no es un estado de 1s. De manera similar, tampoco es en realidad un estado 2p.
Está planteando una pregunta ("Me pregunto si...") sin ningún esfuerzo por investigar una respuesta. Para alguien con una reputación tan alta, es un mal ejemplo para los recién llegados. Además, este no es un sitio de discusión.
No me preocupa el ejemplo que doy a los recién llegados.
En la teoría de la perturbación dependiente del tiempo, para describir la probabilidad de transición entre los estados propios de energía, se utiliza la dependencia temporal del cuadrado de las funciones propias de energía superpuestas (correspondientes al momento dipolar oscilante) para calcular la probabilidad de transición. Consulte los libros de texto de QM.
Entonces, ¿el cálculo semiclásico da la misma respuesta o no? Publica una respuesta si puedes.
@Marty Green: el cálculo semiclásico de las probabilidades de transición bajo una perturbación externa no utiliza explícitamente la pérdida o ganancia de energía por radiación. Da la probabilidad de transición en función del tiempo. Además, no existe una teoría semiclásica para la emisión espontánea, excepto la probabilidad derivada de los coeficientes de Einstein.
Cuando dije "el cálculo semiclásico", no me refería al de los libros de texto. Me refiero al que describí en la pregunta. La emisión espontánea no tiene nada que ver con eso porque especifiqué la superposición particular. Y el cálculo que describí específicamente requería el uso de la pérdida de radiación para ajustar los coeficientes s vs p en el curso de la transición.
@Marty Green: creo que su pregunta es interesante, pero no conozco ningún cálculo "semiclásico" del tiempo de descomposición relacionado con la pérdida de energía debido a la emisión de ondas electromagnéticas. En la imagen habitual de la mecánica cuántica, también existe el problema de qué energía asignar a una superposición arbitraria de dos estados propios de energía. La superposición con coeficientes complejos C 1 y C 2 solo da las probabilidades | C 1 | 2 y | C 2 | 2 para medir las energías mi 1 y mi 2 , respectivamente, en este estado de superposición.
@freecharly Me alegra que encuentres mi pregunta interesante. Esperaba que mi intención fuera clara de que el cálculo debería llevarse a cabo asumiendo que la energía REAL de la superposición debería tomarse como |c1|2*E1 + |c2|2 *E2, con c1 creciendo a expensas de c2 como la radiación se lleva la energía fuera del sistema. Creo que es un cálculo bastante sencillo y me resulta extraño que, suponiendo que tenga conocimientos en este territorio, no sepa que nadie haya publicado un cálculo de este tipo.
@Marty Green: entiendo tu suposición. Probablemente esté en desacuerdo con la QM ortodoxa. No soy un experto profesional ni en la historia ni en la teoría cuántica convencional o sus interpretaciones alternativas. Pero creo que podría haber interpretaciones alternativas a la llamada "interpretación de Copenhague" que podrían estar más cerca de la realidad. Así que mantengo una mente abierta en este asunto.
@MartyGreen ¿Por qué crees que es extraño que no se haya hecho antes? Este no es un modelo natural (porque no predice decaimiento para un estado 2s puro, y postula un mecanismo de acción inversa completamente ad hoc). Dicho esto, de hecho es una pregunta interesante, y hay áreas que usan modelos similares (sin la acción posterior ad hoc), que pueden justificarse bajo algunas condiciones adecuadas. Si tengo tiempo lo escribo.
La superposición de un 2s con un estado 1s es un imán giratorio si los dos estados tienen espines opuestos. Entonces irradia clásicamente.

Respuestas (2)

Sospecho que los resultados serían correctos (al menos, aproximadamente), ya que Barut desarrolló su "electrodinámica de campo propio" (ver, por ejemplo, http://phys.lsu.edu/~jdowling/publications/Barut89b.pdf ) y afirmó resultados muy cerca de los de QED. En la electrodinámica de campo propio, la radiación es creada por la densidad de carga relacionada con la función de onda de forma estándar (para el campo de Dirac).

Me decepciona que nadie en este grupo de discusión haya podido publicar una respuesta definitiva sobre si el cálculo semiclásico, aplicando las ecuaciones de Maxwell a la carga oscilante de la mecánica cuántica, da el resultado correcto para la emisión de radiación de un excitado. átomo de hidrógeno. Agradezco la referencia de akhmeteli a una publicación relacionada, pero no aborda directamente esta pregunta. Por lo tanto, voy a tener que responder esta pregunta lo mejor que pueda demostrando un cálculo de tipo "reverso del sobre".

Dije que quería considerar la superposición 50-50 de los estados 1s y 2p. Primero necesitamos saber el momento dipolar máximo de la superposición. Encontré el resultado en este sitio web de la Universidad de Texas por el Prof. Richard Fitpatrick. Creo que lo estoy interpretando correctamente cuando digo que el desplazamiento máximo de carga es de 0,4 angstroms (alrededor del 75% del radio estándar del estado fundamental).

Entonces necesitamos la frecuencia de la oscilación. Por supuesto, esta es la diferencia de frecuencia correspondiente a la diferencia de energía de 10,5 eV de los estados, o 1,6 x 10^16 rad/seg.

Ahora podemos calcular la aceleración. La forma más fácil de hacer esto es pretender que es un movimiento circular uniforme y usar w^2*r. Obtengo una aceleración de 10^22/m-seg^2. (Dado que en realidad es un movimiento armónico y no circular, esto nos dará un factor de error de 2 en el resultado final).

Ahora conecto esta aceleración en la fórmula de Larmour. Puede encontrar la fórmula de Larmour en cualquier lugar de Internet, pero simplemente he convertido todas las constantes físicas en valores numéricos, y se trata de

   **Radiated Power  =   6 x 10^-54 a^2**

Puede ver que cuando conecto mi valor de aceleración en esta fórmula, obtengo una potencia de radiación total de 6 x 10^-10 vatios. Esto lo dividimos por 2 para tener en cuenta el movimiento armónico frente al circular.

¿Es este el poder correcto? Tenemos que convertir a "tiempo de transición" para averiguarlo. La energía total del estado excitado es 10,5 (llámalo 10) eV, lo que equivale a 1,6 x 10^-18 julios. Dividiendo la energía por la potencia, obtenemos que la vida útil del estado excitado es de unos 5 nanosegundos. O tal vez me equivoque con la energía y debería tomarla como la mitad (debido a la superposición), lo que me daría una vida útil de 2,5 nanosegundos. Algo como eso.

Esto puede no ser exacto, pero creo que es más o menos en el estadio de béisbol.

Es difícil creer lo rápido que llegan los votos negativos en esta respuesta. ¿Hay algo mal con mi física?
Mientras tanto, descubrí que en el capítulo 9.3.2 Vida útil de un estado excitado en el libro de texto de D. Griffith "Introducción a la mecánica cuántica" hay, para el caso de un oscilador armónico, un cálculo de la potencia de radiación emitida según QM y según la electrodinámica clásica.
No debe olvidar que la "vida útil", que también se indica en el sitio web vinculado como 1,6 ns para esta transición, es en realidad el tiempo hasta que un gran número norte 0 de átomos de H excitados ha decaído a norte 0 / mi . Este tiempo de vida, que supone un salto cuántico repentino para la transición, no es el mismo que está calculando con la consideración de energía emitida.
Si es exactamente la misma vida o no, está bastante cerca. Y el "salto cuántico repentino" es, por supuesto, una suposición al igual que mi radiación continua. Ambos supuestos son modelos que nos guían sobre cómo hacer el cálculo, y ninguno parece demostrable experimentalmente frente al otro.
En realidad, debería ser posible diferenciar experimentalmente entre estos modelos. Si el modelo de radiación clásico fuera correcto, entonces todos los átomos deberían decaer al mismo tiempo. Mecánicamente cuántica, algunos decaen mucho antes y otros mucho más tarde. También se ve esto en la desintegración radiactiva. Si bien la vida media le da la descomposición de una gran cantidad de átomos a la mitad, hay muchos átomos que se descomponen solo después de un tiempo de órdenes de magnitud más largos.
Debes pensar con mucho cuidado acerca de cómo configurarías un experimento para demostrar esta supuesta diferencia. No creo que puedas hacerlo.
Busqué en Google brevemente y encontré este artículo sobre mediciones de por vida de estados excitados: slac.stanford.edu/cgi-wrap/getdoc/slac-pub-14754.pdf
Esta respuesta muestra que obtiene respuestas en el orden de magnitud aproximado, pero eso no es tan inesperado como cree: ambas son teorías (relativamente) simples con pocas constantes dimensionales, que se usan de manera relativamente similar, por lo que no hay todo eso mucho espacio para que discrepen. En cualquier caso, te estás perdiendo la parte importante: para que el modelo sea de alguna utilidad, debe predecir directamente una caída exponencial, y no está claro que pueda hacerlo (porque la amplitud de oscilación del dipolo disminuye a medida que la cambios de superposición).
Tampoco sé de dónde viene tu decepción. La pregunta tal como se plantea es interesante, pero requiere un cálculo bastante sustancial, que apenas se explora en la literatura (y por buenas razones), por lo que responder esto lleva tiempo.